what is 0.06 divided by 8 working out

Answers

Answer 1
It’s an answer for your question
What Is 0.06 Divided By 8 Working Out
Answer 2

Answer:

0.0075

Step-by-step explanation:

 0. 0 0 7 5

_______________________

8  ÷ 0. 0 6 0 0

− 0        

_______________________

 0 0      

−   0      

_______________________

   0 6    

 −   0    

_______________________

    6 0  

   − 5 6  

_______________________

      4 0

     − 4 0

_______________________

         0

If my answer is incorrect, pls correct me!

If you like my answer and explanation, mark me as brainliest!

-Chetan K


Related Questions

Solve the system by substitution.
2x – 3y = 38
-x – 6 = y

Answers

Answer:

x=19,0

y=0,-38/3

Step-by-step explanation:

Answer:

x = 4, y = -10

Step-by-step explanation:

-x - 6 - y = 0

-x - y = 6

2x - 3y = 38, -x - y =6

2x - 3y = 38

2x = 3y + 38

Divide both sides by 2

x = 1/2(3y+38)

Multiple 1/2 times 3y + 38

x= 3/2 y + 19

-(3/2y + 19) -y =6

-3/2y - 19 - y = 6

-5/2y -19 = 6

-5/2y = 25

y = -10

x = 3/2(-10) + y

x = -15 + 19

x = 4

x = 4, y = -10

HELLPPPP YALLLLLLLLK ASAPPPPP

Answers

4) Let the number be : N

Given : 10% of this number is 25

[tex]\sf{\implies 10\% \ of \ N = 25}[/tex]

[tex]\sf{\implies \dfrac{10}{100} \times N = 25}[/tex]

[tex]\sf{\implies \dfrac{N}{10} = 25}[/tex]

[tex]\sf{\implies N = 250}[/tex]

Now we need to find 60% of 250

[tex]\sf{\implies \dfrac{60}{100} \times 250}[/tex]

[tex]\sf{\implies 6 \times 25}[/tex]

[tex]\sf{\implies 150}[/tex]

5) Given : James left the home at 10 : 00 AM

First : He will meet his friend Tom at the skate park which is 15 minutes walk from his home.

So, James reached the skate park 10 : 15 AM

Second : They stayed at the skate park for an hour

So, till 11 : 15 AM James and Tom were at skate park

Third : Then they will take a 10 minutes walk over to the Burger bar

So, James and Tom reached the Burger bar at 11 : 25 AM

6) First let us find 75% of 80

[tex]\sf{\implies \dfrac{75}{100} \times 80}[/tex]

[tex]\sf{\implies 15 \times 4}[/tex]

[tex]\sf{\implies 60}[/tex]

Now let us find 40% of 30

[tex]\sf{\implies \dfrac{40}{100} \times 30}[/tex]

[tex]\sf{\implies 12}[/tex]

Now, (60 - 12) = 48

Answer: 75% of 80 is 48 greater than 40% of 30

7) Given : Journey to school takes 70 minutes

We know that : 1 hour = 60 minutes

So, 70 minutes = 1 hour + 10 minutes

Given : Boy leaves the home at 0715

Answer: The Boy arrives at 0825

Congruent angle pairs : Find m

Answers

Answer:

75

Step-by-step explanation:

x+60 and 5x are vertical angles, which means they are equal

x+60 = 5x

Subtract x from each side

x+60-x = 5x-x

60 = 4x

Divide by 4

60/4 = 4x/4

15 =x

We want to find x+60

x+60

15+60

75

Answer:

∠GH = 75°

Step-by-step explanation:

x + 60° = 5x° ( vertical angles )

subtract x from both side

x - x + 60° = 5x - x

60° = 4x

divide each side by 4

60°/ 4 = 4x / 4

15 ° = x

To calculate, ∠GH

∠GH = x + 60 °

15 ° + 60° .. ( x = 15°)

= 75°

Black pens come in boxes of 12. Blue pens come in boxes of 10. Red
pens also come in boxes of 10. You need a total of 50 of each color of
pen. You have 2 boxes of black pens, 3 boxes of blue pens, and 1 box of
red pens. How many boxes of each color should you order?

Answers

Answer

12+10+10+50+2+3+1=88

Step-by-step explanation:

Answer:

Red pens: 4 boxes

blue pens: 2 boxes

black: 3 boxes

Step-by-step explanation:

which point is a solution to the inequality shown in this graph


(0, 5)
(3, 2)
(-3, -6)
(5, 0)

Answers

Answer:

(0,5)

Step-by-step explanation:

if the point is graphed in the blue area then it is a solution to the inequality

Note: because the line is dashed points graphed on the line are not solutions

(0,5) is graphed in the blue region meaning that (0,5) is a solution to the inequality.

even though we may have found the answer let's check the rest just to be safe

(3,2) and (-3,-6) are graphed on the line

remember because the line is dashed points that fall on it are not solutions

therefore (3,2) and (-3,-6) are not Solutions

(5,0) when graphed is not in the shaded area and is therefore not a solution

we can conclude that a is the answer

Write an algorithm to find sum of first ten even numbers?

plz do it ... if you do i will mark brainliest ​

Answers

Answer:

n(n+1)

This is the answer I know

Combine like terms
1) 3y + 4y
2) -7( -4 - 2s)
3) 7f - 9f - 4
4) 9 + 7 + 6d + 2d
5) 7 + 9r + 3
6) -2(7 - 8s) - 4
7) q - 9q
8) -3( -2h + 6)
9) 3q + 9q - 4 + 2
10) 3 + 5(2d - 6)

Answers

Answer:

1.7y

2.-14s

3.-2f

4.23d

5.19r

6.-2s

7.-9q

8.24h

9.10q

10.23d

Step-by-step explanation:

I TRY MY BEST TO SOLVE IT

HOPE IT HELPS

CORRECT ME IF I AM WRONG

1. 7y

2. 28 + 14s

3. -2f - 4

4. 16 + 8d

5. 10 + 9r

6. -18+ 16s

7. -8q

8. 6h - 18

9. 12q - 2

10. 10d -27

SUPER URGENT: What is the range of y = sinx in the interval - π ≤ x ≤ 0?
-1 ≤ y ≤ 1
-1 ≤ y ≤ 0
0 ≤ y ≤ 1
y ≤ 1

Answers

Answer:

-1 ≤ y ≤ 0

Step-by-step explanation:

here, we simply want to know the range of values for sin x over the given interval

The range of values for sin x are simply the values in which y will take over the given interval of x

let’s start with the value sine (-pi)

We have this as;

0

And the value of sin (0) = 0

Recall, between -pi and 0, we have (-pi/2)

so sin (-pi/2) = -1

Thus, we have the correct range of values as;

-1 ≤ y ≤ 0

Tell whether the triangle with the given side lengths is a right triangle. Y/N question

Answers

Answer:

No.

Step-by-step explanation:

33^2 + 56^2 is not equal to 66^2. I squared these numbers because that is how I can find out and see if the Pythagorean Theorem (a^2 + b^2 = c^2) is true for this.

Answer:

no , it is not a right triangle.

Step-by-step explanation:

according to the pythagoras theorem to be the right angled triangle the sum of square of two smaller sides of triangle must be equal to the square of it's hypotenuse.

Here , 66 is largest side so it is hypotenuse and other two sides are the legs of the triangle.

pythagoras theorem:

a^2 + b^2 = c^2

33^2 + 56^2 = 66^2

1089 + 3136 = 4356

4225 = 4356

since both sides are not equal it can be concluded that the given side lengths does not form right triangle.

Omar's gas tank is 1/3 full. after he buys 9 gallons of gas, it is 5/6 full. How many gallons can Omar's tank hold?

Answers

Answer:

18 gallons

Step-by-step explanation:

First get a common denominator (6). So 3/6 are added to the tank when Omar buys 9 gallons of gas. That means for every 1/6, three gallons are added to the tank.

Jim writes 10^4 words every day how many words does he writes in 10^3 days?​

Answers

1,000 words

10^4=10,000
10^3=1,000

Enter the explicit rule for the geometric sequence.

120,40,403,409,4027

Answers

Answer:

120(1/3)^n-1

This is the correct answer for sure.

120(1/3)^n-1


Explanation: just because

6,347.2184 to the nearest thousandth

Answers

Answer: 6347.218

Step-by-step explanation:

Answer:6,347.218

Step-by-step explanation:

look at the thousandths place and then look at the number next to it if the number was 5 and above the answer would be 6,347.219 but in this situation it's not so you would keep everything the same but subtract the 0.0004

Help please! I really need it

Answers

I think it’s none of those.
Radius is a line that reaches the middle of the circle, so like
——
CA

Marie made lasagna for dinner. She ate 1/6 of it, Morris ate 1/3 of it, and Larry ate 1/4 of it. How much of the lasagna was left?

Answers

Answer:

1/12 of the lasagna was left

Step-by-step explanation:

To find out how much of the lasagna has been eaten, you have to find the least common denominator (LCD) for the three fractions. The LCD of 1/3, 1/4, and 1/6 is 12 (3x4x6). Now convert all the fractions so that all of their denominators is equal to 72 (1/3=4/12, 1/4=3/12, 1/6=2/12), then add all of the fractions together (4/12 + 3/12 +2/12 = 3/4). 3/4 of the lasagna has been eaten and the remaining lasagna is 1/12 (1-3/4=4/4-3/4).

Answer:

[tex]\frac{1}{4}[/tex] of the lasagna

Step-by-step explanation:

[tex]\frac{1}{4} +\frac{1}{6}+ \frac{1}{3}[/tex]

4 = 2 × 2

3 = 3

6 = 2 × 3

LCM = 2 × 2 × 3

LCM = 12

4 × 3 × 6 = 72

[tex]\frac{18}{72} +\frac{12}{72}+ \frac{24}{72}[/tex]

[tex]\frac{18+12+24}{72}[/tex]

[tex]\frac{54}{72}[/tex]

54 = 2 × 3 × 3 × 3

72 = 2 × 2 × 2 × 3 × 3

GCF = 2 × 3 × 3

GCF = 18

[tex]\frac{\frac{54}{18} }{\frac{72}{18} }=\frac{3}{4}[/tex]

[tex]1-\frac{3}{4}[/tex]

[tex]\frac{4}{4} -\frac{3}{4}[/tex]

[tex]\frac{4}{4} -\frac{3}{4}=\frac{1}{4}[/tex]

PLEASE HELP! WAYYY OVERDUE!!!
Look at the box plots and decide whether each team's distribution is symmetric or skewed negatively or positively.

Answers

Answers:

Red = negative skewYellow = symmetricBlue = positive skew

================================================

Explanation:

A symmetric distribution is where we have the left half be a mirror copy of the right half. In other words, we have symmetry going on.

For the yellow team, note the distance between Q1 and the median is 1 unit (47-46 = 1), and so is the distance from the median to Q3  (48-47 = 1). So the box portion has perfect symmetry. You should find that the whiskers are also perfectly symmetric as well using a similar technique of finding the distances and comparing them.

Overall, the entire box-and-whisker plot for the yellow team is perfectly symmetric.

---------------

If a distribution isn't symmetric, then it's skewed in some way. If the right side is pulled longer than the left, then we consider it "skewed to the right" aka "positively skewed". So that applies to the blue team. The red team has negative skew because its left tail is longer than the right tail.

03.
(a)
Multiply out
3(2x-7)
Answer

Answers

Answer: -42 is the answer.

A 3-column table with 5 rows. Column 1 is labeled Tickets with entries 2, 4, 6, 8, 10. Column 2 is labeled Total Cost for Aquarium with entries 29, 58, 87, blank, blank. Column 3 is labeled Total Cost for Wave Pool with entries 33.5, 67, blank, blank, blank.
Gale found out that the wave pool has a money-saving deal: if more than 4 people attend, the tickets are discounted from $16.75 each to $12.25 each.

Calculate to determine how much 6 tickets to the wave pool will cost.

$

Answers

Answer

What event will cost the least for 9 tickets?

the wave pool

How much less will this event cost?

$20.25

Hope it helps :)

Step-by-step explanation:

PLEASE HELP ME- i don't know what to do

Answers

Answer:

5x-20=3x(alternate angle)

5x-3x=20

2x=20

x=20/2

x=10

now ,

2y +4y+5x-20=180(sum of angles of a triangle)

6y+50-20=180

6y+30=180

6y=180-30=150

y=150/6

y=25

hope this will help

A line with a slope of 3 passes through the point (2, 6). What is its equation
in slope-intercept form?
please help i need my ixl done and i don’t understand this lesson

Answers

Answer:

I found: y=−3x+12

Explanation:

You can use the relationship:

y−y0=m(x−x0)

Where m is the slope and x0,y0 the coordinates of your point.

So with your data:

y−6=−3(x−2)

y=−3x+6+6

y=−3x+12

or:

3x+y=12

Determine the area of a parallelogram if
its base is 16 cm and its height is 7 cm.

Answers

Answer: 112 square cm

=====================================

Work Shown:

area = base*height

area = 16*7

area = 112 square cm

----------------

Extra info (optional section):

The area formula for a parallelogram is the same as the formula for a rectangle. Imagine tilting a rectangle so that it's slanted. The area has not changed.

Or another thing to imagine is that let's say we had a stack of 2 by 4s. If the stack is perfectly vertical, aka forming a rectangle, then we can nudge the beams so that we can form a slanted parallelogram instead. Both configurations involve the same amount of wood, and therefore would lead to the same frontal surface area.

Oof, I'm to tired to remember how to do this T-T
Can anyone help asap? This is timed. Brainliest for best answer!!

Answers

Answer:

D

Step-by-step explanation:

Equation of the line

y - y1 = m(x - x1)

y - (-4) = 1/3 (x - 3)

y + 4 = 1/3x - 1

y = 1/3 x - 5

the graph that intercept y axis in -5 is the graph D

Answer:

i can't remember how to do this rn because its like 2:30am

C seems like most accurate answer, if this is wrong im really sorry :C

Step-by-step explanation:

A set of weights includes an 8 lb. barbell and 6 pairs of weight plates. Each pair of plates weighs 15 lb. If x pairs of plates are added to the barbell, the total weight of the barbell and plates in pounds can be represented by f(x) = 15x + 8. What is the range of the function for this situation?

Answers

Answer

8,98

Step-by-step explanation:

the minimum amount of weight you can use is the barbell alone which is 8lbs by itself and the maximum is 98 because there are 6 15lb plates which equal to 90lb total and when you add the 6 plates to the barbell you get 98lbs

Find the measure of AB

A. 20
B. 21
C. 22
D. 14

Answers

Answer:

[tex]CB.CA=CD .CK[/tex]

[tex]9(9+2x+5)=10(10+2x+1)[/tex]

[tex]9(2x+14)=10(2x+11)[/tex]

[tex]18x+126=20x+110[/tex]

[tex]126-110=20x-18x[/tex]

[tex]16=2x->x=8[/tex]

[tex]AB=2(18)+5[/tex]

[tex]=21[/tex]

So, your answer is B) 21

hope it helps...

Please I need help with this!!!!!!!

Answers

Answer:

Step-by-step explanation:

1). Step 4:

   [tex]x=5^{\frac{4}{3}}=(5^4)^{\frac{1}{3}}[/tex]

   [tex]x=\sqrt[3]{5^4}[/tex] [Since, [tex]a^{\frac{1}{3}}=\sqrt[3]{a}[/tex]]

   [tex]x=\sqrt[3]{5\times 5\times 5\times 5}[/tex]

   Step 5:

   [tex]x=\sqrt[3]{(5)^3\times 5}[/tex]

   [tex]x=\sqrt[3]{5^3}\times \sqrt[3]{5}[/tex]

2). He simplified the expression by removing exponents from the given expression.

3). Let the radical equation is,

   [tex](3x-1)^{\frac{1}{5}}=2[/tex]

   Step 1:

   [tex](3x-1)^{\frac{1}{5}\times \frac{5}{1} }=2^{\frac{5}{1}}[/tex]

   Step 2:

   [tex](3x-1)=2^5[/tex]

   Step 3:

   [tex]3x=32+1[/tex]

   Step 4:

   [tex]x=11[/tex]

4). By substituting [tex]x=11[/tex] in the original equation.

   [tex](3\times 11-1)^{\frac{1}{5}}=(32)^\frac{1}{5}[/tex]

                         [tex]=(2^5)^\frac{1}{5}[/tex]

                         [tex]=2[/tex]

There is no extraneous solution.

1). Step 4:

   x=5^{\frac{4}{3}}=(5^4)^{\frac{1}{3}}x=534=(54)31

   x=\sqrt[3]{5^4}x=354 [Since, a^{\frac{1}{3}}=\sqrt[3]{a}a31=3a

   x=\sqrt[3]{5\times 5\times 5\times 5}x=35×5×5×5

   Step 5:

   x=\sqrt[3]{(5)^3\times 5}x=3(5)3×5

   x=\sqrt[3]{5^3}\times \sqrt[3]{5}x=353×35

2). He simplified the expression by removing exponents from the given expression.

3). Let the radical equation is,

   (3x-1)^{\frac{1}{5}}=2(3x−1)51=2

   Step 1:

   (3x-1)^{\frac{1}{5}\times \frac{5}{1} }=2^{\frac{5}{1}}(3x−1)51×15=215

   Step 2:

   (3x-1)=2^5(3x−1)=25

   Step 3:

   3x=32+13x=32+1

   Step 4:

   x=11x=11

4). By substituting x=11x=11 in the original equation.

   (3\times 11-1)^{\frac{1}{5}}=(32)^\frac{1}{5}(3×11−1)51=(32)51

                         =(2^5)^\frac{1}{5}=(25)51

                         =2=2

There is no extraneous solution.

A teacher writes the following product on the board:
(5x + 2)(x + 4) = 5x^2 +22x +8
Palesa says that 5x^2 +22x + 8 is a factor of 5x +2.
Nune says that 5x+ 2 is divisible by 5x^2 +22x +8.
Who is correct?

Answers

This is obvious come on Joe is correct

Help please help please

Answers

Answer:

12.57 in⠀⠀⠀⠀⠀

Answer:

12.56 inches

Step-by-step explanation:

A= pie x r^2

= 3.14 × 2^2

= 3.14 × 4

= 12.56

[tex]\sqrt \frac{256}{4\\}[/tex]

Answers

Answer:

8

Step-by-step explanation:

[tex]\sqrt{\frac{256}{4} }[/tex]

[tex]\sqrt{\frac{16*16}{2*2} }[/tex]

[tex]\sqrt{(\frac{16}{2})^2 }[/tex]

square and root gets cancel . so ,

[tex]\frac{16}{2}[/tex]

8

what is the volume of the following rectangular prism 9/5 units 2/3 units ​

Answers

Answer:

1.2

Step-by-step explanation:

9/5*2/3=1.2

Need answer fasttt
Find the volume of a pyramid with a square base, where the side length of the base is
11.5 in and the height of the pyramid is 6.7 in. Round your answer to the nearest
tenth of a cubic inch.

Answers

Answer:

295.4 [tex]in^3[/tex]

Step-by-step explanation:

[tex]V=\frac{1}{3}(b)^2(h)\\\\ =\frac{1}{3} (11.5)^2(6.7)\\\\=\frac{1}{3}(135.25)(6.7)\\\\[/tex]

= 295.358

≈ 295.4

Other Questions
What was Jean-Jacques Rousseau's contribution to the Enlightenment duringthe 17th century?A. His theory of separation of powers formed the basis for France'snew government.B. He was the charismatic military leader of the rebels during theFrench Revolution.C. He mentored Napoleon Bonaparte, convincing him to experimentwith democracy.D. His ideas on government influenced how people believed ademocracy should function. A company operates three machines during three shifts each day. From production records, the data in the table below were collected. At the .05 level of significance test to determine if the number of breakdowns is independent of the shift. Machine Shift A B C1 Observed 46 11 13 Expected 40.27933 14.07821 15.642462 Observed 37 10 11 Expected 33.3743 11.6648 12.960893 Observed 20 15 16 Expected 29.34637 10.25698 11.39665a. Yes, you can reject the claim that the number of breakdowns is independent of the shift because the p-value = 0.0423b. Yes, you can reject the claim that the number of breakdowns is independent of the shift because the p-value = 0.9577c. No, you cannot reject the claim that the number of breakdowns is independent of the shift because the p-value = 0.0423d. No, you cannot reject the claim that the number of breakdowns is independent of the shift because the p-value = 0.9577 In rhombus LMNO, diagonals top enclose L N end enclose and top enclose M O end enclose intersect at P. If the measure of angle L P M is represented by 2x + 70. Solve for x. Consider the video and advertisements that you've seen. Now, write a script to advertise any product of your choosing. The product can be real or fictitious. Use figurative language as a rhetorical device to enhance the persuasiveness of your advertisement. Also, consider your audience. To improve on your advertisement, start by creating an outline, then review and revise your first draft. If you have access to recording equipment, you may also try to record and present your advertisement as a radio spot. Liam spent $4.76 on a salad bowl, and $3.81 on a cup of coffee. He paid using TWO ten dollar bills. What was Liam's change? What was the effect of the battle of gettysburg on troop strength five adaptive features of animals in forest habitat A specific component for a warm up and cool down for running would be _____? A. volleying for serve B. lay ups C. jogging in place D. stationary bike riding Can someone help me with this one PLEASE HELP!! ILL GIVE MORE POINTS TO THE FASTEST ANSWER Solve y = x + 6 for x.Ox = y + 6Ox= -y + 6O x = y - 6Ox= -y-6 please help thank you Find the length of TU. Read the excerpt.After a strenuous climb, the hikers decided to make camp before reaching the summit because night was approaching rapidly.Which statement most accurately describes this excerpt?It contains two dependent clauses and one independent clause. It contains one independent clause.It contains one dependent clause and one independent clause.It contains one independent clause and two dependent clauses. According to your textbookSelect one:a. all motivation comes from the desire for external rewardsb. today's motivation researchers emphasize the biological drives thatguide behaviorc. human motivation is based entirely on psychological goals, notbiological needsd. drive theories do not account for the full complexity of humanmotivation Identifying Changes of MatterWhich of these are chemical changes in matter? Check all tha Why did European explorers land in the Americas? a. They wanted to establish a mercantile economic system b. They wanted to find gold and other precious metals and gems c. They believed they had reached India d. They were looking to conquer Native Americans nadaot ang inyong lugar sa usa ka kusog na bagyo pahayag na may paglaom Is the triangle a right angle? Pythagorean Theorem. Out of 350 applicants for a job, 158 are female and 66 are female and have a graduate degree. Step 2 of 2 : If 110 of the applicants have graduate degrees, what is the probability that a randomly chosen applicant is female, given that the applicant has a graduate degree